4

Problem. I am attempting to find a generalized result for the sum $$S(a, b) = \displaystyle\sum_{k=a}^b \sqrt{k},$$ i.e. a function (= closed expression in elementary functions) $T(a, b)$ such that $$\lfloor S(a, b) \rfloor = \lfloor T(a, b) \rfloor$$ for all $0 < a < b$.

This question is inspired from a Romanian math olympiad problem:

Let $N= \sqrt{5} + \sqrt{6} + \sqrt{7} + \cdots + \sqrt{13}$. Without approximating the square roots, show that the whole part of $N$ is 26.

A proof idea I had was to use the Euler-Maclaurin formula, and then adding enough terms to bound the error uniformly over all $a, \, b$. The formula is:

$$ \displaystyle\sum_{k=a}^b f(k) = \displaystyle\int\limits_a^b f(x) \; \mathrm dx + \frac{f(b) + f(a)}{2} + \sum_{k=1}^{\left\lfloor \frac{p}{2}\right\rfloor} \frac{B_{2k}}{(2k)!} \left(f^{(2k - 1)}(b) - f^{(2k - 1)}(a)\right) + R_p.$$

Here $B_k$ is the $k$th Bernoulli number with $B_1 = 1/2$ and $R_p$ is an error term. The error term can be expressed exactly in terms of the Bernoulli polynomials.

These polynomials are recursively defined by: $$B_0(x) = 1, \quad B_k'(x) = k B_{k-1}(x), \quad \displaystyle\int\limits_0^1 B_k(x) \; \mathrm dx = 0.$$

The periodized Bernoulli functions are:

$$P_k(x) = B_k(x - \lfloor x \rfloor), \quad \text{for } x \in [0,1).$$

The remainder term in the Euler-Maclaurin formula is: $$R_p(a, b) = (-1)^{p+1} \int\limits_a^b \frac{f^{(p)}(x) \, P_p(x)}{p!} \; \mathrm dx.$$

Using the bound on Bernoulli polynomials $$|B_k(x)| \leq \displaystyle\frac{2 k!}{(2\pi)^k} \zeta(k), \quad \text{for } k > 0,$$ we get an estimate for the remainder $$|R_p(a, b)| \leq \frac{2 \zeta(p)}{(2\pi)^p} \displaystyle\int\limits_m^n |f^{(p)}(x)| \; \mathrm dx.$$

The idea is pretty straightforward: use $f(x) = \sqrt{x}$ and enough terms to guarantee that $\lfloor S(a, b) \rfloor = \lfloor T(a, b) \rfloor$.

The first order approximation

$$T_1(a, b) = \left\lfloor \frac{2}{3}(b^{3/2} - a^{3/2}) + \frac{\sqrt{a} + \sqrt{b}}{2} \right\rfloor$$

is not good enough. For example

$$\displaystyle \sum_{k=1}^{49} \sqrt{k} \approx 231.96473279…$$

so $\lfloor S(1, 49) \rfloor = 231$, while $T_1(1, 49) =232$.

Trying to add one more term (so that we get the second order Euler-Maclaurin approximation),

$$T_2(a, b) = \frac{2}{3}\,\left(b^{3/2}-a^{3/2}\right) + \frac{\sqrt a+\sqrt b}{2} + \frac{1}{24}\,\left(\frac1{\sqrt b}-\frac1{\sqrt a}\right)$$

yields a failure at $(a, b) = (1, 156)$ because

$$\displaystyle \sum_{k=1}^{156} \sqrt{k} \approx 1305.00003144…$$

but $T_2(1, 156) \approx 1304.99958433...$, so the floors are not equal.

I could numerically confirm an alternating pattern: odd orders fail at $(1, 156)$, even orders at $(1,761)$. This alternation persists at least up to order 30.

Since the formula is so close, can we add a correction term somehow or prove that enough terms will give the result, i.e. that

$$\lfloor S(a, b) \rfloor = \lfloor T_n(a, b) \rfloor$$

for all $0 < a < b$?

This answer to a similar question says it is enough "to take more terms", but does not offer any proof for that claim.

River Li
  • 49,125
  • My thought would be to analyze the function $f(x)=\sqrt {9+x}+\sqrt {9-x}$. Your sum is $3+f(1)+f(2)+f(3)+f(4)$, $f(0)=6$ and $f(x)$ is decreasing. so you just need to show that $f(4)$ isn't too small. Numerically, it seems the function is very well approximated in that range by its second order Taylor series which, if true, makes the problem easy enough but of course you still need to bound the error. – lulu Jun 27 '25 at 13:42
  • Possibly worth remarking: sticking with my notation, a cheap thought would be to remark that $27>S>3+4f(5)$, where $S$ denotes the desired sum. Even if you can compute $f(5)$ sharply though, the right hand is $3+4\times 5.741657387=25.96662955$ which is just a hair too small to settle the point. – lulu Jun 27 '25 at 14:26
  • 2
    Why not round to the nearest integer instead of taking the floor? – Qiaochu Yuan Jun 27 '25 at 16:09
  • 1
    Does knowing that $2< \sqrt 5 < 3, 2<\sqrt 6 < 3... ,3< \sqrt 13< 4$ etc count as "approximating the square roots"? – fleablood Jun 27 '25 at 19:20
  • I found that I have learned the trick in your question, that is, to use telescoping series to approximate the sum. See e.g. https://math.stackexchange.com/questions/4656181/special-sum-to-show-by-hand-se/4656256#4656256. – River Li Jun 28 '25 at 02:21
  • Another way is to split the sum into two, e.g. $\sum_{k=1}^{156} \sqrt{k} = \sum_{k=1}^{78} \sqrt{k} + \sum_{k=79}^{156} \sqrt{k}$. – River Li Jun 28 '25 at 02:26
  • For the general case, it seems that using Euler-Maclaurin summation even to very high order we always have a problem at some points. – Claude Leibovici Jun 29 '25 at 05:25

1 Answers1

4

The Olympiad question is easy if you use the fact that the square root function is (strictly) concave.

So the average of the nine values is $<$ the middle value, which is $3$; therefore their sum is $< 27$.

And the average of the nine values is $>$ the average of the two extremes $\sqrt 5$ and $\sqrt{13}$; therefore their sum is $>S=\frac92(\sqrt 5+\sqrt{13})$. Now $S^2=\frac{81}{4}(5+13+2\sqrt{65})>\frac{81}{4}(5+13+16)=\frac{1377}{2}$, and this is $>26^2=676$.

The fact that this lets you pinpoint the integral part of the sum is rather lucky, and won't work in general. So I doubt that your question as stated has a simple solution.

TonyK
  • 68,059